Đến nội dung

Hình ảnh

Đa thức + Số học

- - - - -

  • Please log in to reply
Chủ đề này có 2 trả lời

#1
sinh vien

sinh vien

    Thượng sĩ

  • Thành viên
  • 260 Bài viết

Bài toán . ( Putnam 2011 ) Cho p là một số nguyên tố  lẻ .

Chứng minh rằng có ít nhất $\frac{p+1}{2}$ giá trị $n\in \left \{ 0,1,2... p-1 \right \}$ sao cho $\sum_{k=0}^{p-1}k!n^{k}$ không chia hết cho p.

Lời giải. Đặt $f(x)=\sum_{k=0}^{p-1}k!x^{k}\in F_{p}[x]$ là tập hợp các đa thức bậc p có hệ số thuộc trường $F_{p}$. Ta phân hoạch $F_{p}=Z\bigcup Z^{'}$, 

 trong đó $Z=\left \{ a\in F_{p};f(a)=0 \right \};Z^{'}=\left \{ a\in F_{p} ;f(a)\neq 0\right \}$

 Ta thấy $f(0)=1\Rightarrow 0\in Z^{'}\Rightarrow \left | Z \right |\geqslant 1$.

 Đặt $\left | Z \right |=p-r,\left | Z^{'} \right |=r\geqslant 1$.

 Xét đa thức $s(x)=\prod_{a\in Z^{'}}(x-a)\Rightarrow degs(x)=r$

   Dựa vào các ta định nghĩa thì đa thức $s(x)f(x)$ triệt tiêu ( luôn bằng 0 ) tại mọi điểm của trường $F_{p}$,

Do đó $s(x)f(x)=(x^{p}-x)h(x)$, trong đó h(x) có bậc r-1 và thuộc $F_{p}[x]$ ( Lưu ý đến bậc của hai vế + Định lý Fermat nhỏ ). Trong những tính toán tiếp theo có sử dụng đến đạo hàm hình thức .

$x\frac{d}{dx}(xf(x))=x\frac{d}{dx}\sum_{k=0}^{p-1}k!x^{k+1}=x\sum_{k=0}^{p-2}(k+1)!x^{k}=\sum_{k=1}^{p-1}k!x^{k}=f(x)-1$

$\Rightarrow x^{2}f^{'}(x)+xf(x)=f(x)-1\Rightarrow x^{2}f^{'}(x)+(x-1)f(x)+1=0$

  Lấy đạo hàm hình thức $s(x)f(x)=(x^{p}-x)h(x)$ ta được

      $s^{'}(x)f(x)+s(x)f^{'}(x)=\left ( x^{p}-x \right )h^{'}(x)-h(x)$ ( Chú ý loại bỏ các số hạng triệt tiêu trên trường $F_{p}$ )

 Từ các kết luận trên ta thấy

 $x^{2}s^{'}(x)f(x)=(x^{p}-x)x^{2}h^{'}(x)-x^{2}h(x)-s(x)x^{2}f^{'}(x)$

                          $=(x^{p}-x)x^{2}h^{'}(x)-x^{2}h(x)+s(x)((x-1)f(x)+1)$

                         $=(x^{p}-x)x^{2}h^{'}(x)-x^{2}h(x)+(x-1)(x^{p}-x)h(x)+s(x)$

                         $=\left ( x^{p}-x \right )(x^{2}h^{'}(x)-(x-1)h(x))-x^{2}h(x)+s(x)$

Ta thấy vế trái triệt tiêu trên $Z\bigcup \left \{ 0 \right \}$,  $x^{p}-x$ triệt tiêu trên $F_{p}$ nên đa thức $-x^{2}h(x)+s(x)$ triệt tiêu trên $Z\bigcup\left \{ 0 \right \}$.

Ta thấy

 $r+1=deg(-x^{2}h(x)+s(x))\geqslant \left | Z \right |+1=p-r+1\Rightarrow r\geqslant \frac{p+1}{2}$


Bài viết đã được chỉnh sửa nội dung bởi sinh vien: 14-05-2015 - 08:32


#2
sinh vien

sinh vien

    Thượng sĩ

  • Thành viên
  • 260 Bài viết

Bài toán (Putnam 2008 ) Cho p là một số nguyên tố . Gỉa sử h(x) là một đa thức hệ số nguyên sao cho $\left \{ h(0),h(1),...,h(p^{2}-1) \right \}$ là một hệ thặng dư đầy đủ modulo $p^{2}$.

 Chứng minh rằng $\left \{ h(0),h(1),...,h(p^{3}-1) \right \}$ lập thành một hệ thặng dư đầy đủ modulo $p^{3}$



#3
sinh vien

sinh vien

    Thượng sĩ

  • Thành viên
  • 260 Bài viết

Bài toán (Putnam 1977 ) Chứng minh rằng $C_{pa}^{pb}\equiv C_{a}^{b}(mod p)$, với mọi số nghuyên dương a, b và $a\geq b$ , p là số nguyên tố

 Lời giải. Trong lời giải có đề cập đến một số kết quả cơ bản về đa thức trên vành $Z_{p}$

  Theo một kết quả quen thuộc $C_{p}^{i}\equiv 0(modp),i=\overline{1,p-1}$.

Phát biểu lại kết quả trên bằng ngôn ngữ đại số ta được : $(1+x)^{p}=1+x^{p}$.

 Ta thấy

     $\sum_{k=0}^{pa}C_{pa}^{k}x^{k}=(1+x)^{pa}=\left [ (1+x)^{p} \right ]^{a}=(1+x^{p})^{a}=\sum_{j=0}^{a}C_{a}^{j}x^{jp}$

Đồng nhất hệ số của $x^{pb}$ ở hai vế ta thu được

                 $C_{pa}^{pb}\equiv C_{a}^{b}(modp)$

 Lưu ý rằng các tính toán đa thức của ta ở đây đang thực hiện trên $Z_{p}[x]$


Bài viết đã được chỉnh sửa nội dung bởi sinh vien: 14-05-2015 - 17:54





0 người đang xem chủ đề

0 thành viên, 0 khách, 0 thành viên ẩn danh